CR weaken 'except' question

This topic has expert replies
Senior | Next Rank: 100 Posts
Posts: 44
Joined: Fri Mar 02, 2012 10:50 am

CR weaken 'except' question

by bhopalkararpit » Fri Jul 20, 2012 10:19 pm
A recent survey found that more computers than copies of computer programs were purchased by Germans last year. The best interpretation of this finding is that the practice of illegally duplicating commercial computer programs is widespread among Germans.
Each of the following, if true, would counter some reasonable objection to the interpretation advanced above EXCEPT:

(A) Few German computer users write their own computer programs.
(B) There are few free noncommercial computer programs available in Germany.
(C) Some Germans purchase computers outside of Germany for use in Germany, and such purchases were counted in the survey.
(D) The typical German computer user has several commercially written computer programs on his or her computer.
(E) Many Germans legally make duplicates of commercial computer programs, but such duplicates were counted as purchased copies in the survey.

Master | Next Rank: 500 Posts
Posts: 156
Joined: Sun Jul 15, 2012 12:03 pm
Location: New York, USA
Thanked: 34 times
Followed by:1 members

by kartikshah » Sat Jul 21, 2012 5:27 am
The answer should be C

A. If few write programs on their own then they needn't buy. This is a reasonable objection. Eliminate
B. If free programs are available, the Germans needn't buy programs. This is a reasonable objection. Eliminate
C. Even if they purchase from outside Germany, they would still need computer programs. No explanation given for not purchasing a computer program. Seems like the correct choice. Retain for now.
D. If they already have computer programs on their computer, why would they buy? This is a reasonable objection. Eliminate
E. If they make legal copies and if these legal copies were accounted as purchase in the survey, where's the problem? This is a reasonable objection. Eliminate

So the best option should be C

User avatar
Legendary Member
Posts: 520
Joined: Sat Apr 28, 2012 9:12 pm
Thanked: 339 times
Followed by:49 members
GMAT Score:770

by eagleeye » Sat Jul 21, 2012 1:14 pm
bhopalkararpit wrote:A recent survey found that more computers than copies of computer programs were purchased by Germans last year. The best interpretation of this finding is that the practice of illegally duplicating commercial computer programs is widespread among Germans.
Each of the following, if true, would counter some reasonable objection to the interpretation advanced above EXCEPT:
What a question! Such a convoluted stem. Let's figure it out.

Argument Premise: More computers were bought by Germans than computer program copies.
Argument Conclusion: Germans must be making a lot of illegal copies.

Now the question stem. Phew! Someone must have really wanted to mess with the reader's head. ;)
We'll tackle it anyway:

"Each of the following, if true, would counter some reasonable objection to the interpretation advanced above EXCEPT:"

1. COUNTER SOME REASONABLE OBJECTION TO THE INTERPRETATION ADVANCED ABOVE.
The interpretation advanced above = the conclusion.
Some reasonable objection to the conclusion = some statement that would weaken the conclusion.
"Countering" such statement would be a statement that weakens a reasonable weakening statement. A double negative which tells us that we need to find something that strengthens the conclusion. But wait, we have part 2:

2. EXCEPT. Hence we need to find a statement that DOES NOT STRENGTHEN the conclusion. So we are going to find 4 statements that sort of STRENGTHEN the argument and one that doesn't.

(A) Few German computer users write their own computer programs.
If only few of them write their own programs, then the conclusion is stronger that they are making illegal copies. INCORRECT. (This one counters the objection "But a lot of German computer users write their own computer programs).

(B) There are few free noncommercial computer programs available in Germany.
Same as above. If there are only few free non-commercial computer programs available, many users might still be using the illegal copies of commercial programs. This one mildly strengthens the argument. INCORRECT. (This one counters the objection "But there are a lot of free non-commercial programs that Germans can buy").

(C) Some Germans purchase computers outside of Germany for use in Germany, and such purchases were counted in the survey.
The argument premise only talks about whether the Germans bought the computers. There is no mention of "where" the computers were bought. This is irrelevant to the argument and hence it does not strengthen. CORRECT.

(D) The typical German computer user has several commercially written computer programs on his or her computer.
Again, if a typical user has more than one commercially written computer programs, and the conclusion says that less people bought the software than the number of computers, there may be some strong illegal copying going on. This one strengthens too. INCORRECT. (This one counters the objection, "But Germans don't even have a single commercial program on their computers")

(E) Many Germans legally make duplicates of commercial computer programs, but such duplicates were counted as purchased copies in the survey.
Again, this brings the total number of purchased copies to an even smaller number, thereby strengthening the argument. INCORRECT. (This one counters the objection, "But Many Germans make legal copies of programs, it's not illegal duplication then")

C is the correct answer for this WEAKEN the WEAKENER, EXCEPT question.

Let me know if this helps :)

Senior | Next Rank: 100 Posts
Posts: 44
Joined: Fri Mar 02, 2012 10:50 am

by bhopalkararpit » Sat Jul 21, 2012 9:50 pm
Thanks a lot...yes certainly your ans helps :)